48÷2(9+3) = ???

Originally Posted by do work son

Originally Posted by ServeChilled81

I think we see who the real troll is here.
Like I said, Danica Mckellar has actual accredited work in this very field of mathematics
for me, it's enough to satisfy an end to this matter.

send her the OG problem with a division sign instead of a / and see if she interprets it the same way

she wont change her answer, why do you guys fail to realize that its the same thing?

as if x is different from *
30t6p3b.gif

Code:
 
Originally Posted by do work son

Originally Posted by ServeChilled81

I think we see who the real troll is here.
Like I said, Danica Mckellar has actual accredited work in this very field of mathematics
for me, it's enough to satisfy an end to this matter.

send her the OG problem with a division sign instead of a / and see if she interprets it the same way

she wont change her answer, why do you guys fail to realize that its the same thing?

as if x is different from *
30t6p3b.gif

Code:
 
Originally Posted by do work son

Originally Posted by ServeChilled81

I think we see who the real troll is here.
Like I said, Danica Mckellar has actual accredited work in this very field of mathematics
for me, it's enough to satisfy an end to this matter.

send her the OG problem with a division sign instead of a / and see if she interprets it the same way

This.
 
Originally Posted by do work son

Originally Posted by ServeChilled81

I think we see who the real troll is here.
Like I said, Danica Mckellar has actual accredited work in this very field of mathematics
for me, it's enough to satisfy an end to this matter.

send her the OG problem with a division sign instead of a / and see if she interprets it the same way

This.
 
Originally Posted by Gameover2

Originally Posted by do work son

Originally Posted by ServeChilled81

I think we see who the real troll is here.
Like I said, Danica Mckellar has actual accredited work in this very field of mathematics
for me, it's enough to satisfy an end to this matter.

send her the OG problem with a division sign instead of a / and see if she interprets it the same way

This.
co-sign. I think people are thrown off by the division sign because you rarely see it in higher level mathematics and then automatically convert it to "/" on paper or in their head
 
Originally Posted by Gameover2

Originally Posted by do work son

Originally Posted by ServeChilled81

I think we see who the real troll is here.
Like I said, Danica Mckellar has actual accredited work in this very field of mathematics
for me, it's enough to satisfy an end to this matter.

send her the OG problem with a division sign instead of a / and see if she interprets it the same way

This.
co-sign. I think people are thrown off by the division sign because you rarely see it in higher level mathematics and then automatically convert it to "/" on paper or in their head
 
The answer is 288. The people who are getting 2 as the answer don't realize that the order of operations (PEMDAS) indicates that Multiplication and Division have the same rank and that Addition and Subtraction have the same rank, therefore Multiplication doesn't always come before Division, the proper way to solve a problem is from left to right when you have multiples of the same rank. PEMDAS could just as easily be written PEDMSA and we would get the same results because of the left to right rule.

" Parentheses outrank exponents, which outrank multiplication and division (but multiplication and division are at the same rank), and these two outrank addition and subtraction (which are together on the bottom rank). When you have a bunch of operations of the same rank, you just operate from left to right. "

http://www.purplemath.com/modules/orderops.htm
 
Originally Posted by do work son

Originally Posted by ServeChilled81

I think we see who the real troll is here.
Like I said, Danica Mckellar has actual accredited work in this very field of mathematics
for me, it's enough to satisfy an end to this matter.

send her the OG problem with a division sign instead of a / and see if she interprets it the same way

Ya'll really trying to say that the SIGN dictates the answer to the problem now? 
roll.gif
roll.gif
roll.gif
roll.gif
roll.gif

Wow..... I can't..... I just can't anymore. You're really gonna sit there and tell me that the division sign and the / aren't the same damn thing?

I'm out. It's been real. 
 
The answer is 288. The people who are getting 2 as the answer don't realize that the order of operations (PEMDAS) indicates that Multiplication and Division have the same rank and that Addition and Subtraction have the same rank, therefore Multiplication doesn't always come before Division, the proper way to solve a problem is from left to right when you have multiples of the same rank. PEMDAS could just as easily be written PEDMSA and we would get the same results because of the left to right rule.

" Parentheses outrank exponents, which outrank multiplication and division (but multiplication and division are at the same rank), and these two outrank addition and subtraction (which are together on the bottom rank). When you have a bunch of operations of the same rank, you just operate from left to right. "

http://www.purplemath.com/modules/orderops.htm
 
Originally Posted by do work son

Originally Posted by ServeChilled81

I think we see who the real troll is here.
Like I said, Danica Mckellar has actual accredited work in this very field of mathematics
for me, it's enough to satisfy an end to this matter.

send her the OG problem with a division sign instead of a / and see if she interprets it the same way

Ya'll really trying to say that the SIGN dictates the answer to the problem now? 
roll.gif
roll.gif
roll.gif
roll.gif
roll.gif

Wow..... I can't..... I just can't anymore. You're really gonna sit there and tell me that the division sign and the / aren't the same damn thing?

I'm out. It's been real. 
 
Originally Posted by JChambers

The answer is 288. The people who are getting 2 as the answer don't realize that the order of operations (PEMDAS) indicates that Multiplication and Division have the same rank and that Addition and Subtraction have the same rank, therefore Multiplication doesn't always come before Division, the proper way to solve a problem is from left to right when you have multiples of the same rank. PEMDAS could just as easily be written PEDMSA and we would get the same results because of the left to right rule.

" Parentheses outrank exponents, which outrank multiplication and division (but multiplication and division are at the same rank), and these two outrank addition and subtraction (which are together on the bottom rank). When you have a bunch of operations of the same rank, you just operate from left to right. "

http://www.purplemath.com/modules/orderops.htm

look at the 1st example on the second page of that article. it proves why the answer is 2
 
Originally Posted by JChambers

The answer is 288. The people who are getting 2 as the answer don't realize that the order of operations (PEMDAS) indicates that Multiplication and Division have the same rank and that Addition and Subtraction have the same rank, therefore Multiplication doesn't always come before Division, the proper way to solve a problem is from left to right when you have multiples of the same rank. PEMDAS could just as easily be written PEDMSA and we would get the same results because of the left to right rule.

" Parentheses outrank exponents, which outrank multiplication and division (but multiplication and division are at the same rank), and these two outrank addition and subtraction (which are together on the bottom rank). When you have a bunch of operations of the same rank, you just operate from left to right. "

http://www.purplemath.com/modules/orderops.htm

look at the 1st example on the second page of that article. it proves why the answer is 2
 
whoevers messaging winnie cooper, ask her if parenthesis in PEMDAS only applies to equations actually inside the parentheses

that should settle everything
 
whoevers messaging winnie cooper, ask her if parenthesis in PEMDAS only applies to equations actually inside the parentheses

that should settle everything
 
The answer is 2. The entire bracket equation MUST be done first. 2(9+3) is 24


You don't just add 9 + 3 then work left to right. The entire 2(9+3) is done first.
 
The answer is 2. The entire bracket equation MUST be done first. 2(9+3) is 24


You don't just add 9 + 3 then work left to right. The entire 2(9+3) is done first.
 
Originally Posted by do work son

Originally Posted by JChambers

The answer is 288. The people who are getting 2 as the answer don't realize that the order of operations (PEMDAS) indicates that Multiplication and Division have the same rank and that Addition and Subtraction have the same rank, therefore Multiplication doesn't always come before Division, the proper way to solve a problem is from left to right when you have multiples of the same rank. PEMDAS could just as easily be written PEDMSA and we would get the same results because of the left to right rule.

" Parentheses outrank exponents, which outrank multiplication and division (but multiplication and division are at the same rank), and these two outrank addition and subtraction (which are together on the bottom rank). When you have a bunch of operations of the same rank, you just operate from left to right. "

http://www.purplemath.com/modules/orderops.htm

look at the 1st example on the second page of that article. it proves why the answer is 2
That ain't prove @#$# man. It proves that you go from LEFT to RIGHT.
 
Originally Posted by do work son

Originally Posted by JChambers

The answer is 288. The people who are getting 2 as the answer don't realize that the order of operations (PEMDAS) indicates that Multiplication and Division have the same rank and that Addition and Subtraction have the same rank, therefore Multiplication doesn't always come before Division, the proper way to solve a problem is from left to right when you have multiples of the same rank. PEMDAS could just as easily be written PEDMSA and we would get the same results because of the left to right rule.

" Parentheses outrank exponents, which outrank multiplication and division (but multiplication and division are at the same rank), and these two outrank addition and subtraction (which are together on the bottom rank). When you have a bunch of operations of the same rank, you just operate from left to right. "

http://www.purplemath.com/modules/orderops.htm

look at the 1st example on the second page of that article. it proves why the answer is 2
That ain't prove @#$# man. It proves that you go from LEFT to RIGHT.
 
Originally Posted by ncmalko1

The answer is 2. The entire bracket equation MUST be done first. 2(9+3) is 24


You don't just add 9 + 3 then work left to right. The entire 2(9+3) is done first.
That's one thing they refuse to understand.
 
Originally Posted by ncmalko1

The answer is 2. The entire bracket equation MUST be done first. 2(9+3) is 24


You don't just add 9 + 3 then work left to right. The entire 2(9+3) is done first.
That's one thing they refuse to understand.
 
Originally Posted by CertifiedSW

Originally Posted by do work son

Originally Posted by JChambers

The answer is 288. The people who are getting 2 as the answer don't realize that the order of operations (PEMDAS) indicates that Multiplication and Division have the same rank and that Addition and Subtraction have the same rank, therefore Multiplication doesn't always come before Division, the proper way to solve a problem is from left to right when you have multiples of the same rank. PEMDAS could just as easily be written PEDMSA and we would get the same results because of the left to right rule.

" Parentheses outrank exponents, which outrank multiplication and division (but multiplication and division are at the same rank), and these two outrank addition and subtraction (which are together on the bottom rank). When you have a bunch of operations of the same rank, you just operate from left to right. "

http://www.purplemath.com/modules/orderops.htm

look at the 1st example on the second page of that article. it proves why the answer is 2
That ain't prove @#$# man. It proves that you go from LEFT to RIGHT.
Wrong. You have to do 2(9+3) first.
 
Originally Posted by CertifiedSW

Originally Posted by do work son

Originally Posted by JChambers

The answer is 288. The people who are getting 2 as the answer don't realize that the order of operations (PEMDAS) indicates that Multiplication and Division have the same rank and that Addition and Subtraction have the same rank, therefore Multiplication doesn't always come before Division, the proper way to solve a problem is from left to right when you have multiples of the same rank. PEMDAS could just as easily be written PEDMSA and we would get the same results because of the left to right rule.

" Parentheses outrank exponents, which outrank multiplication and division (but multiplication and division are at the same rank), and these two outrank addition and subtraction (which are together on the bottom rank). When you have a bunch of operations of the same rank, you just operate from left to right. "

http://www.purplemath.com/modules/orderops.htm

look at the 1st example on the second page of that article. it proves why the answer is 2
That ain't prove @#$# man. It proves that you go from LEFT to RIGHT.
Wrong. You have to do 2(9+3) first.
 
Originally Posted by kingcrux31

Originally Posted by ncmalko1

The answer is 2. The entire bracket equation MUST be done first. 2(9+3) is 24


You don't just add 9 + 3 then work left to right. The entire 2(9+3) is done first.
That's one thing they refuse to understand.

no, what you guys refuse to understand is that THERE ARE NO BRACKETS, youre placing them in your mind to help you (mis)understand the problem instead of taking problem as it is

no unproven brackets... no unproven parentheses... take the problem as it is
 
Back
Top Bottom